Skip to main content

Theory and Modern Applications

q-Hardy type inequalities for quantum integrals

Abstract

The aim of this work is to obtain quantum estimates for q-Hardy type integral inequalities on quantum calculus. For this, we establish new identities including quantum derivatives and quantum numbers. After that, we prove a generalized q-Minkowski integral inequality. Finally, with the help of the obtained equalities and the generalized q-Minkowski integral inequality, we obtain the results we want. The outcomes presented in this paper are q-extensions and q-generalizations of the comparable results in the literature on inequalities. Additionally, by taking the limit \(q\rightarrow 1^{-}\), our results give classical results on the Hardy inequality.

1 Introduction

Hardy’s integral inequality, proved by G.H. Hardy in 1920 [4] is

$$ \int _{0}^{\infty } \biggl( \frac{1}{x} \int _{0}^{x}f ( t ) \,dt \biggr) ^{p}\,dx\leq \biggl( \frac{p}{p-1} \biggr) ^{p} \int _{0}^{ \infty }f^{p} ( t ) \,dt, $$
(1.1)

where \(p>1\), \(x>0\), f is a nonnegative measurable function on \(( 0,\infty ) \) and \(\int _{0}^{\infty }f^{p} ( t ) \,dt\) is convergent. Also the constant \(( \frac{p}{p-1} ) ^{p}\) is the best possible.

Hardy’s type inequalities have been studied by a large number of authors during the 20th century and has motivated some important lines of study which are currently active. Over the last 20 years a large number of papers have appeared in the literature which deal with the simple proofs, various generalizations and discrete analogues of Hardy’s inequality and its generalizations; see [5, 8, 11, 12, 15, 1719].

The inequalities have become an important cornerstone in mathematical analysis and optimization and many uses of these inequalities have been discovered in a variety of settings. Recently, the Hermite–Hadamard type inequality has become the subject of intensive research. For recent results, refinements, counterparts, generalizations and new Hadamard’s-type inequalities, see [1, 7, 10, 14, 16, 20].

On the other hand, the study of calculus without limits is known as quantum calculus or q-calculus. The famous mathematician Euler initiated the study q-calculus in the 18th century by introducing the parameter q in Newton’s work of infinite series. In the early 20th century, Jackson [6] has started a symmetric study of q-calculus and introduced q-definite integrals. The subject of quantum calculus has numerous applications in various areas of mathematics and physics, such as number theory, combinatorics, orthogonal polynomials, basic hyper-geometric functions, quantum theory, mechanics and in theory of relativity. This subject has received outstanding attention by many researchers and hence it is considered as an in-corporative subject between mathematics and physics. The reader is referred to [2, 3, 9] for some current advances in the theory of quantum calculus and theory of inequalities in quantum calculus.

The purpose of this work is to establish quantum estimates for q-Hardy type integral inequalities on quantum calculus. For this, we establish new identities including quantum derivatives and quantum numbers. After that, we prove a generalized q-Minkowski integral inequality. Finally, with the help of the obtained equalities and the generalized q-Minkowski integral inequality, we obtain the results we want. The outcomes presented in this paper are q-extensions and q-generalizations of the comparable results in the literature on inequalities. In addition, by taking the limit \(q\rightarrow 1^{-}\), our results give classical results on the Hardy inequality.

2 Preliminaries and definitions of q-calculus

Throughout this paper, let \(a< b\) and \(0< q<1\) be a constant. The following definitions, notations and theorems for q-derivative and q-integral of a function f on \([ a,b ]\) are given in [2, 3, 9].

The notation \([ z ] _{q}\) is defined by

$$ [ z ] _{q}=\frac{1-q^{n}}{1-q}\quad \bigl( z\in \mathbb{C} ;q\in \mathbb{C} \backslash \{ 1 \} ;q^{z}\neq 1 \bigr). $$
(2.1)

A special case of (2.1) when \(z\in \mathbb{N} \) is

$$ [ n ] _{q}=\frac{1-q^{n}}{1-q}=1+q+q^{2}+\cdots +q^{n-1} \quad ( n\in \mathbb{N} ). $$

Also

$$ [ -n ] _{q}=-\frac{1}{q^{n}} [ n ] _{q} \quad ( n\in \mathbb{N} ). $$
(2.2)

Definition 1

Let \(f: [ a,b ] \rightarrow \mathbb{R} \) be a continuous function, then q-derivative of f at \(x\in [ a,b ] \) is characterized by the expression

$$ D_{q}f ( x ) = \frac{f ( x ) -f ( qx ) }{ ( 1-q ) x},\quad x\neq 0. $$
(2.3)

Since \(f: [ a,b ] \rightarrow \mathbb{R} \) is a continuous function, thus we have \(D_{q}f ( a ) =\underset{x\rightarrow a}{\lim }D_{q}f ( x ) \) The function f is said to be q- differentiable on \([ a,b ] \) if \(D_{q}f ( t ) \) exists for all \(x\in [ a,b ] \). Also \(\underset{q\rightarrow 1^{-}}{\lim }D_{q}f ( x ) =f^{\prime } ( x ) \) is classic derivative.

Theorem 1

Assume that \(f,g:I\subset \mathbb{R} \rightarrow \mathbb{R} \) are continuous functions, then we have the properties of the q-derivative:

$$\begin{aligned} (\mathrm{I}) &\quad D_{q} \bigl( af ( x ) \pm bg ( x ) \bigr) =aD_{q}f ( x ) \pm bD_{q}g ( x ) . \\ (\mathrm{II}) &\quad D_{q} \bigl( f ( x ) g ( x ) \bigr) =f ( qx ) D_{q}g ( x ) +g ( x ) D_{q}f ( x ) . \\ (\mathrm{III}) &\quad D_{q} \biggl( \frac{f ( x ) }{g ( x ) } \biggr) = \frac{f ( qx ) D_{q}g ( x ) +g ( x ) D_{q}f ( x ) }{g ( x ) g ( qx ) }. \end{aligned}$$

Definition 2

Suppose \(0< a< b\). The definite q-integral is defined as

$$ \int _{0}^{b}f ( t ) \,d_{q}t= ( 1-q ) b \sum_{n=0}^{\infty }q^{n}f \bigl( q^{n}b \bigr) $$
(2.4)

and

$$ \int _{a}^{b}f ( t ) \,d_{q}t= \int _{0}^{b}f ( t ) \,d_{q}t- \int _{0}^{a}f ( t ) \,d_{q}t, $$

where \(\sum_{n=0}^{\infty }q^{n}f ( q^{n}b ) \) and \(\sum_{n=0}^{\infty }q^{n}f ( q^{n}a ) \) are convergent.

Definition 3

([9])

The improper q-integral of \(f ( t ) \) on \([ 0,\infty ) \) is defined by

$$ \int _{0}^{\infty }f ( t ) \,d_{q}t=\sum _{n=- \infty }^{\infty } \int _{q^{n+1}}^{q^{n}}f ( t ) \,d_{q}t= ( 1-q ) \sum_{n=-\infty }^{\infty }q^{n}f \bigl( q^{n} \bigr) \quad ( 0< q< 1 ) $$

and

$$ \int _{0}^{\infty }f ( t ) \,d_{q}t=\sum _{n=- \infty }^{\infty } \int _{q^{n}}^{q^{n+1}}f ( t ) \,d_{q}t= \frac{q-1}{q}\sum_{n=-\infty }^{\infty }q^{n}f \bigl( q^{n} \bigr) \quad ( 1< q ) , $$

where \(\sum_{n=-\infty }^{\infty }q^{n}f ( q^{n} ) \) is convergent.

We have the following properties of the q-integral of (2.4):

$$\begin{aligned} (\mathrm{I}) &\quad D_{q} \int _{a}^{x}f ( t ) \,d_{q}t=f ( x ) . \\ (\mathrm{II}) &\quad \int _{a}^{x}D_{q}f ( t ) \,d_{q}t=f(x)-f(a). \\ (\mathrm{III}) & \quad \int _{a}^{x} \bigl[ f ( t ) \pm g ( t ) \bigr] \,d_{q}t= \int _{a}^{x}f ( t ) \,d_{q}t \pm \int _{a}^{x}g ( t ) \,d_{q}t. \\ (\mathrm{IV}) &\quad \int _{0}^{x}t^{\alpha }\,d_{q}t= \frac{x^{\alpha +1}}{ [ \alpha +1 ] _{q}}, \quad \text{for }\alpha \in \mathbb{R} \backslash \{ -1 \} . \\ (\mathrm{V}) & \quad \text{The integration by parts rule of the }q\text{-integral:} \\ &\quad \int _{c}^{x}f ( t ) D_{q}g ( t ) \,d_{q}t= f ( t ) g ( t ) \vert _{c}^{x}- \int _{c}^{x}g ( qt ) D_{q}f ( t ) \,d_{q}t. \end{aligned}$$
(2.5)

Theorem 2

(q-Hölder inequality)

Let f, g be q-integrable on \([ a,b ] \) and \(0< q<1\) and \(\frac{1}{s}+\frac{1}{r}=1\) with \(s>1\). Then we have

$$ \int _{a}^{b} \bigl\vert f ( t ) g ( t ) \bigr\vert \,d_{q}t\leq \biggl( \int _{a}^{b} \bigl\vert f ( t ) \bigr\vert ^{s}\,d_{q}t \biggr) ^{\frac{1}{s}} \biggl( \int _{a}^{b} \bigl\vert f ( t ) g ( t ) \bigr\vert ^{r}\,d_{q}t \biggr) ^{\frac{1}{r}}. $$

3 Auxiliary results

The following results which will be used. There is no general change of variables property for the q-integral. However, the variable can be changed as follows.

Lemma 1

(q-Change of variables property)

Let \(f:I\rightarrow \mathbb{R} \) be a function and \(0< q<1\). Then we have

$$ \int _{0}^{1}f ( sb ) \,d_{q}s= \frac{1}{b} \int _{0}^{b}f ( t ) \,d_{q}t, $$
(3.1)

where \(b\neq 0\) and \(\int _{0}^{b}f ( t ) \,d_{q}t\) is convergent.

Proof

From the definition of the q-integral, we have

$$\begin{aligned}& \int _{0}^{1}f ( sb ) \,d_{q}s \\& \quad = ( 1-q ) ( 1-0 ) \sum_{n=0}^{\infty }q^{n}f \bigl( \bigl[ q^{n}1+ \bigl( 1-q^{n} \bigr) 0 \bigr] b \bigr) \\& \quad = \frac{1}{b} \int _{0}^{b}f ( t ) \,d_{q}t \end{aligned}$$

as desired. □

A general chain rule for q-derivative does not exist. However, a chain rule of \(( h ( t ) ) ^{p}\) and \(( h ( t ) ) ^{\frac{1}{p}}\) can be calculated as follows.

Lemma 2

Let \(h:I\subset \mathbb{R} \rightarrow \mathbb{R} \) be a function \(p\in \mathbb{Z} \) and \(0< q<1\). Then we have

$$ D_{q} \bigl( h ( t ) \bigr) ^{p}= \Biggl( \sum _{i=0}^{p-1} \bigl[ h ( t ) \bigr] ^{p-1-i} \bigl[ h ( qt ) \bigr] ^{i} \Biggr) D_{q}h ( t ) . $$
(3.2)

In (3.2) if we choose \(q\rightarrow 1^{-}\) we have the classical derivative of \(( h ( t ) ) ^{p}\),

$$ \lim_{q\rightarrow 1^{-}}D_{q} \bigl( h ( t ) \bigr) ^{p}=p \bigl( h ( t ) \bigr) ^{p-1}h^{\prime } ( t ) = \bigl[ \bigl( h ( t ) \bigr) ^{p} \bigr] ^{\prime }. $$

Proof

By the definition of the q-derivative we have

$$\begin{aligned}& D_{q} \bigl( h ( t ) \bigr) ^{p} \\& \quad = \frac{ [ h ( t ) ] ^{p}- [ h ( qt ) ] ^{p}}{ ( 1-q ) t} \\& \quad = \frac{ ( h ( t ) -h ( qt ) ) }{ ( 1-q ) t}\sum_{i=0}^{p-1} \bigl[ h ( t ) \bigr] ^{p-1-i} \bigl[ h ( qt ) \bigr] ^{i} \\& \quad = \Biggl( \sum_{i=0}^{p-1} \bigl[ h ( t ) \bigr] ^{p-1-i} \bigl[ h ( qt ) \bigr] ^{i} \Biggr) D_{q}h ( t ) \end{aligned}$$

as desired. □

Lemma 3

Let \(h:I\subset \mathbb{R} \rightarrow \mathbb{R} \) be a function \(p\in \mathbb{Z} \) and \(0< q<1\). Then we have

$$ D_{q} \bigl( h ( t ) \bigr) ^{\frac{1}{p}}= \frac{D_{q}h ( t ) }{\sum_{i=0}^{p-1} ( h ( t ) ) ^{\frac{p-1-i}{p}} ( h ( qt ) ) ^{\frac{i}{p}}} . $$
(3.3)

In (3.3) if we choose \(q\rightarrow 1^{-}\) we have the classical derivative of \(( h ( t ) ) ^{\frac{1}{p}}\),

$$ \lim_{q\rightarrow 1^{-}}D_{q} \bigl( h ( t ) \bigr) ^{ \frac{1}{p}}=\frac{h^{\prime } ( t ) }{p ( h ( t ) ) ^{\frac{p-1}{p}}}= \bigl[ \bigl( h ( t ) \bigr) ^{\frac{1}{p}} \bigr] ^{\prime }. $$

Proof

We consider

$$\begin{aligned}& y ( t ) = \bigl( h ( t ) \bigr) ^{ \frac{1}{p}}, \\& \bigl( y ( t ) \bigr) ^{p} = \bigl( h ( t ) \bigr), \end{aligned}$$

such that

$$ D_{q} \bigl( y ( t ) \bigr) ^{p}=D_{q} \bigl( h ( t ) \bigr), $$
(3.4)

and from (3.2) we know

$$ D_{q} \bigl( y ( t ) \bigr) ^{p}= \Biggl( \sum _{i=0}^{p-1} \bigl[ y ( t ) \bigr] ^{p-1-i} \bigl[ y ( qt ) \bigr] ^{i} \Biggr) D_{q}y ( t ) =D_{q} \bigl( h ( t ) \bigr) . $$
(3.5)

Thus, we get

$$ D_{q}y ( t ) = \frac{D_{q}h ( t ) }{\sum_{i=0}^{p-1} ( h ( t ) ) ^{\frac{p-1-i}{p}} ( h ( qt ) ) ^{\frac{i}{p}}} $$

as desired. □

Similarly, we have more general result as follows.

Lemma 4

Let \(h:I\subset \mathbb{R} \rightarrow \mathbb{R} \) be a function \(\frac{n}{m}\in \mathbb{Q} \) and \(0< q<1\). Then we have

$$ D_{q} \bigl( h ( t ) \bigr) ^{\frac{n}{m}}= \frac{\sum_{i=0}^{n-1} ( h ( t ) ) ^{n-1-i} ( h ( qt ) ) ^{i}}{\sum_{i=0}^{m-1} ( h ( t ) ) ^{\frac{n ( m-1-i ) }{m}} ( h ( qt ) ) ^{\frac{ni}{m}}}D_{q}h ( t ). $$
(3.6)

In (3.6) if we choose \(q\rightarrow 1^{-}\) we have the classical derivative of \(( h ( t ) ) ^{\frac{n}{m}}\),

$$ \lim_{q\rightarrow 1^{-}}D_{q} \bigl( h ( t ) \bigr) ^{ \frac{n}{m}}=\frac{n}{m} \bigl( h ( t ) \bigr) ^{\frac{n}{m}-1}h^{ \prime } ( t ) = \bigl[ \bigl( h ( t ) \bigr) ^{\frac{n}{m}} \bigr] ^{\prime }. $$

Proof

We consider

$$\begin{aligned}& y ( t ) = \bigl( h ( t ) \bigr) ^{ \frac{n}{m}} , \\& \bigl( y ( t ) \bigr) ^{m} = \bigl( h ( t ) \bigr) ^{n}, \end{aligned}$$

such that

$$ D_{q} \bigl( y ( t ) \bigr) ^{m}=D_{q} \bigl( h ( t ) \bigr) ^{n}, $$

and from (3.2) we have

$$\begin{aligned}& \Biggl( \sum_{i=0}^{m-1} \bigl[ y ( t ) \bigr] ^{m-1-i} \bigl[ y ( qt ) \bigr] ^{i} \Biggr) D_{q}y ( t ) \\& \quad = \Biggl( \sum_{i=0}^{n-1} \bigl[ h ( t ) \bigr] ^{n-1-i} \bigl[ h ( qt ) \bigr] ^{i} \Biggr) D_{q} \bigl( h ( t ) \bigr) . \end{aligned}$$

Thus, we get

$$\begin{aligned} D_{q}y ( t ) =&\frac{\sum_{i=0}^{n-1} [ h ( t ) ] ^{n-1-i} [ h ( qt ) ] ^{i}}{\sum_{i=0}^{m-1} [ y ( t ) ] ^{m-1-i} [ y ( qt ) ] ^{i}}D_{q}h ( t ) \\ =&\frac{\sum_{i=0}^{n-1} ( h ( t ) ) ^{n-1-i} ( h ( qt ) ) ^{i}}{\sum_{i=0}^{m-1} ( h ( t ) ) ^{\frac{n ( m-1-i ) }{m}} ( h ( qt ) ) ^{\frac{ni}{m}}}D_{q}h ( t ) \end{aligned}$$

as desired. □

4 Main results

Firstly, we will prove the generalized q-Minkokski type integral inequality which will be used in the next theorem.

Theorem 3

(Generalized q-Minkowski integral inequality)

Let \(\alpha \in ( 0,1 ] \), \(1\leq p\leq \infty \), \(f: [ a,b ] \times [ c,d ] \rightarrow \mathbb{R} \) be a q-integrable function. Then the following inequality holds:

$$ \biggl( \int _{a}^{b} \biggl\vert \int _{c}^{d}f ( x,y ) \,d_{q}y \biggr\vert ^{p}\,d_{q}x \biggr) ^{\frac{1}{p}}\leq \int _{c}^{d} \biggl( \int _{a}^{b} \bigl\vert f ( x,y ) \bigr\vert ^{p}\,d_{q}x \biggr) ^{\frac{1}{p}}\,d_{q}y, $$
(4.1)

where \(q\in ( 0,1 ) \).

Proof

The case \(p=1\) corresponds to Fubini’s theorem. For the case \(p=\infty \) we just notice that

$$ \biggl( \int _{a}^{b} \biggl\vert \biggl( \int _{c}^{d}f ( x,y ) \,d_{q}y \biggr) _{q} \biggr\vert ^{p}\,d_{q}x \biggr) ^{ \frac{1}{p}}\leq \int _{c}^{d}\operatorname{ess}\sup_{x\in \mathbb{R} ^{n}} \bigl\vert f ( x,y ) \bigr\vert \,d_{q}y. $$

Now assume that \(1< p<\infty \) and we can write

$$\begin{aligned}& \int _{a}^{b} \biggl\vert \int _{c}^{d}f ( x,y ) \,d_{q}y \biggr\vert ^{p}\,d_{q}x \\& \quad = \int _{a}^{b} \biggl\vert \int _{c}^{d}f ( x,y ) \,d_{q}y \biggr\vert ^{p-1} \biggl\vert \int _{c}^{d}f ( x,y ) \,d_{q}y \biggr\vert \,d_{q}x \\& \quad \leq \int _{a}^{b} \biggl\vert \int _{c}^{d}f ( x,t ) \,d_{q}t \biggr\vert ^{p-1} \biggl( \int _{c}^{d} \bigl\vert f ( x,y ) \bigr\vert \,d_{q}y \biggr) \,d_{q}x \\& \quad = \int _{a}^{b} \biggl( \int _{c}^{d} \biggl\vert \int _{c}^{d}f ( x,t ) \,d_{q}t \biggr\vert ^{p-1} \bigl\vert f ( x,y ) \bigr\vert \,d_{q}y \biggr) \,d_{q}x \\& \quad = \int _{c}^{d} \biggl( \int _{a}^{b} \biggl\vert \int _{c}^{d}f ( x,t ) \,d_{q}t \biggr\vert ^{p-1} \bigl\vert f ( x,y ) \bigr\vert \,d_{q}x \biggr) \,d_{q}y \end{aligned}$$

the last step coming from Fubini’s theorem. By applying the q-Hölder inequality to the inner integral with respect to x, we have

$$\begin{aligned}& \int _{a}^{b} \biggl\vert \int _{c}^{d}f ( x,y ) \,d_{q}y \biggr\vert ^{p}\,d_{q}x \\& \quad \leq \int _{c}^{d} \biggl\{ \biggl( \int _{a}^{b} \biggl( \biggl\vert \int _{c}^{d}f ( x,t ) \,d_{q}t \biggr\vert ^{r ( p-1 ) }\,d_{q}x \biggr) ^{\frac{1}{r}} \biggl( \int _{a}^{b} \bigl\vert f ( x,y ) \bigr\vert ^{p}\,d_{q}x \biggr) ^{ \frac{1}{p}} \biggr) \biggr\} \,d_{q}y \\& \quad = \int _{c}^{d} \biggl\{ \biggl( \int _{a}^{b} \biggl( \biggl\vert \int _{c}^{d}f ( x,t ) \,d_{q}t \biggr\vert ^{p}\,d_{q}x \biggr) ^{\frac{1}{r}} \biggl( \int _{a}^{b} \bigl\vert f ( x,y ) \bigr\vert ^{p}\,d_{q}x \biggr) ^{\frac{1}{p}} \biggr) \biggr\} \,d_{q}y \\& \quad = \biggl( \int _{a}^{b} \biggl\vert \int _{c}^{d}f ( x,t ) \,d_{q}t \biggr\vert ^{p}\,d_{q}x \biggr) ^{\frac{1}{r}} \int _{c}^{d} \biggl( \int _{a}^{b} \bigl\vert f ( x,y ) \bigr\vert ^{p}\,d_{q}x \biggr) ^{\frac{1}{p}}\,d_{q}y. \end{aligned}$$

Finally dividing both sides by \(\int _{a}^{b} ( \vert \int _{c}^{d}f ( x,t ) \,d_{q}t \vert ^{p}\,d_{q}x ) ^{\frac{1}{r}}\) we have

$$ \biggl( \int _{a}^{b} \biggl\vert \int _{c}^{d}f ( x,y ) \,d_{q}y \biggr\vert ^{p}\,d_{q}x \biggr) ^{1-\frac{1}{r}}\leq \int _{c}^{d} \biggl( \int _{a}^{b} \bigl\vert f ( x,y ) \bigr\vert ^{p}\,d_{q}x \biggr) ^{\frac{1}{p}}\,d_{q}y $$

i.e.

$$ \biggl( \int _{a}^{b} \biggl\vert \int _{c}^{d}f ( x,y ) \,d_{q}y \biggr\vert ^{p}\,d_{q}x \biggr) ^{\frac{1}{p}}\leq \int _{c}^{d} \biggl( \int _{a}^{b} \bigl\vert f ( x,y ) \bigr\vert ^{p}\,d_{q}x \biggr) ^{\frac{1}{p}}\,d_{q}y, $$

which gives the required inequality. □

Theorem 4

(q-Hardy inequality)

If f is a nonnegative function on \(( 0,\infty )\), \(p>1\) and \(\int _{0}^{\infty }f^{p} ( t ) \,d_{q}t\) is convergent, then the following inequality holds:

$$ \biggl( \int _{0}^{\infty } \biggl( \frac{1}{x} \int _{0}^{x}f ( t ) \,d_{q}t \biggr) ^{p}\,d_{q}x \biggr) ^{\frac{1}{p}}\leq \frac{1}{ [ \frac{p-1}{p} ] _{q}} \biggl( \int _{0}^{\infty }f^{p} ( t ) \,d_{q}t \biggr) ^{\frac{1}{p}}, $$
(4.2)

where \(q\in ( 0,1 ) \).

Proof

From (3.1) by the q-changing variables \(t=xs\) it follows that

$$ \frac{1}{x} \int _{0}^{x}f ( t ) \,d_{q}t= \int _{0}^{1}f ( sx ) \,d_{q}s. $$

Thus, we write

$$ \biggl( \int _{0}^{\infty } \biggl( \frac{1}{x} \int _{0}^{x}f ( t ) \,d_{q}t \biggr) ^{p}\,d_{q}x \biggr) ^{\frac{1}{p}}= \biggl( \int _{0}^{\infty } \biggl( \int _{0}^{1}f ( xs ) \,d_{q}s \biggr) ^{p}\,d_{q}x \biggr) ^{\frac{1}{p}}. $$
(4.3)

From the generalized q-Minkowski integral inequality and by using the q-changing variables \(xs=t\), we have

$$\begin{aligned}& \biggl( \int _{0}^{\infty } \biggl( \int _{0}^{1}f ( xs ) \,d_{q}s \biggr) ^{p}\,d_{q}x \biggr) ^{\frac{1}{p}} \\& \quad \leq \int _{0}^{1} \biggl( \int _{0}^{\infty }f^{p} ( xs ) \,d_{q}x \biggr) ^{\frac{1}{p}}\,d_{q}s= \int _{0}^{1} \biggl( \int _{0}^{ \infty }\frac{1}{s}f^{p} ( t ) \,d_{q}t \biggr) ^{\frac{1}{p}}\,d_{q}s \\& \quad = \biggl( \int _{0}^{1}s^{-\frac{1}{p}}\,d_{q}s \biggr) \biggl( \int _{0}^{ \infty }f^{p} ( t ) \,d_{q}t \biggr) ^{\frac{1}{p}}= \frac{1}{ [ 1-\frac{1}{p} ] _{q}} \biggl( \int _{0}^{\infty }f^{p} ( t ) \,d_{q}t \biggr) ^{\frac{1}{p}} \end{aligned}$$
(4.4)

from (4.3) and (4.4)

$$ \biggl( \int _{0}^{\infty } \biggl( \frac{1}{x} \int _{0}^{x}f ( t ) \,d_{q}t \biggr) ^{p}\,d_{q}x \biggr) ^{\frac{1}{p}}\leq \frac{1}{ [ \frac{p-1}{p} ] _{q}} \biggl( \int _{0}^{\infty }f^{p} ( t ) \,d_{q}t \biggr) ^{\frac{1}{p}} $$

and the proof is completed. □

Remark 1

In (4.2) if we choose \(q\rightarrow 1^{-}\) we recapture the classical Hardy inequality.

The following theorem generalizes the q-Hardy type integral inequality by introducing power weights \(x^{r}\).

Theorem 5

If f is a nonnegative function on \(( 0,\infty )\), \(p\geq 1\), \(r< p-1\) and \(\int _{0}^{\infty }t^{r}f^{p} ( t ) \,d_{q}t\) is convergent, then the following inequality holds:

$$ \int _{0}^{\infty } \biggl( \frac{1}{x} \int _{0}^{x}f ( t ) \,d_{q}t \biggr) ^{p}x^{r}\,d_{q}x\leq \frac{1}{ [ \frac{p-r-1}{p} ] _{q}^{p}} \int _{0}^{\infty }t^{r}f^{p} ( t ) \,d_{q}t, $$

where \(q\in ( 0,1 ) \).

Proof

By the q-changing variables \(t=xs\) we get

$$ \biggl( \int _{0}^{\infty } \biggl( \frac{1}{x} \int _{0}^{x}f ( t ) \,d_{q}t \biggr) ^{p}x^{r}\,d_{q}x \biggr) ^{\frac{1}{p}}= \biggl( \int _{0}^{\infty } \biggl( \int _{0}^{1}f ( xs ) x^{ \frac{r}{P}}\,d_{q}s \biggr) ^{p}\,d_{q}x \biggr) ^{\frac{1}{p}}. $$

So, from Minkowski q-integral inequality and by the changing variables \(xs=u\) the proof is completed as follows:

$$\begin{aligned}& \biggl( \int _{0}^{\infty } \biggl( \int _{0}^{1}f ( xs ) x^{ \frac{r}{P}}\,d_{q}s \biggr) ^{p}\,d_{q}x \biggr) ^{\frac{1}{p}} \\& \quad \leq \int _{0}^{1} \biggl( \int _{0}^{\infty }x^{r}f^{p} ( xs ) \,d_{q}x \biggr) ^{\frac{1}{p}}\,d_{q}s= \int _{0}^{1} \biggl( \int _{0}^{\infty }\frac{u^{r}}{s^{r+1}}f^{p} ( u ) \,d_{q}u \biggr) ^{ \frac{1}{p}}\,d_{q}s \\& \quad = \biggl( \int _{0}^{1}s^{\frac{-r-1}{p}}\,d_{q}s \biggr) \biggl( \int _{0}^{\infty }u^{r}f^{p} ( u ) \,d_{q}u \biggr) ^{ \frac{1}{p}} \\& \quad = \frac{1}{ [ \frac{p-r-1}{p} ] _{q}} \biggl( \int _{0}^{ \infty }u^{r}f^{p} ( u ) \,d_{q}u \biggr) ^{\frac{1}{p}}. \end{aligned}$$

 □

Remark 2

In Theorem 5 if we put \(r=0\) we obtain the inequality (4.2).

Definition 4

For a given weight r, we define the modified q-Hardy operator as

$$ H_{q,r}f ( x ) =\frac{1}{xr ( x ) } \int _{0}^{x}r ( t ) f ( t ) \,d_{q}t. $$

The following theorem will be proved using the q-Hardy operator.

Theorem 6

Assume f is a nonnegative function on \(( 0,\infty ) \), r being an absolutely continuous function on \(( 0,\infty )\), and \(p>1\). Also assume \(\int _{0}^{\infty }f^{p} ( x ) \,d_{q}x\) is convergent, and

$$ \frac{ [ p-1 ] _{q}}{p}+\frac{x}{p} \frac{D_{q}r ( x ) }{r ( qx ) }\sum _{i=0}^{p-1} \biggl[ \frac{h_{r,a}f ( qx ) }{h_{r,a}f ( x ) } \biggr] ^{i}\geq \frac{1}{\lambda }, $$
(4.5)

for almost every \(x>0\) and for some \(\lambda >0\). Then we have the following inequality:

$$ \int _{0}^{\infty } \bigl( H_{r}f ( x ) \bigr) ^{p}\,d_{q}x \leq \lambda ^{p}\beta ^{p} \int _{0}^{\infty }f^{p} ( x ) \,d_{q}x, $$

where

$$ H_{q,r}f ( x ) =\frac{1}{xr ( x ) } \int _{0}^{x}r ( t ) f ( t ) \,d_{q}t. $$

Proof

We assume \(0< a< b<\infty \) and

$$ h_{q,r,a}f ( x ) =\frac{1}{r ( x ) } \int _{a}^{x}r ( t ) f ( t ) \,d_{q}t. $$

Then, defining \(H_{r,a}f ( x ) =\frac{1}{x}h_{r,a}f ( x ) \), and integrating by parts from (2.5) with \(w= ( h_{r,a}f ( x ) ) ^{p}\) and \(D_{q}g ( x ) =x^{-p}\) noting that \(g(x)=\frac{x^{1-p}}{ [ 1-p ] _{q}}\), we get

$$\begin{aligned}& \int _{a}^{b} \bigl( H_{q,r,a}f ( x ) \bigr) ^{p}\,d_{q}x \\& \quad = \int _{a}^{b} \bigl( h_{q,r,a}f ( x ) \bigr) ^{p}x^{-p}\,d_{q}x \\& \quad = \int _{0}^{b} \bigl( h_{q,r,a}f ( x ) \bigr) ^{p}x^{-p}\,d_{q}x- \int _{0}^{a} \bigl( h_{q,r,a}f ( x ) \bigr) ^{p}x^{-p}\,d_{q}x \\& \quad = \int _{0}^{b} \bigl( h_{q,r,a}f ( x ) \bigr) ^{p}D_{q}\frac{x^{1-p}}{ [ 1-p ] _{q}}\,d_{q}x- \int _{0}^{a} \bigl( h_{q,r,a}f ( x ) \bigr) ^{p}D_{q} \frac{x^{1-p}}{ [ 1-p ] _{q}}\,d_{q}x \\& \quad = \bigl( h_{q,r,a}f ( x ) \bigr) ^{p} \frac{x^{1-p}}{ [ 1-p ] _{q}}\bigg\vert _{0}^{b}- \int _{0}^{b} \frac{ ( qx ) ^{1-p}}{ [ 1-p ] _{q}}D_{q} \bigl( h_{q,r,a}f ( x ) \bigr) ^{p}\,d_{q}x \\& \qquad {} - \bigl( h_{q,r,a}f ( x ) \bigr) ^{p} \frac{x^{1-p}}{ [ 1-p ] _{q}}\bigg\vert _{0}^{a}+ \int _{0}^{a} \frac{ ( qx ) ^{1-p}}{ [ 1-p ] _{q}}D_{q} \bigl( h_{q,r,a}f ( x ) \bigr) ^{p}\,d_{q}x \\& \quad = \bigl( h_{q,r,a}f ( b ) \bigr) ^{p} \frac{b^{1-p}}{ [ 1-p] _{q}} \\& \qquad {} -\frac{q^{1-p}}{ [ 1-p ] _{q}} \int _{0}^{b}x^{1-p}D_{q}h_{q,r,a}f ( x ) \Biggl( \sum_{i=0}^{p-1} \bigl[ h_{q,r,a}f ( x ) \bigr] ^{p-1-i} \bigl[ h_{q,r,a}f ( qx ) \bigr] ^{i} \Biggr) \,d_{q}x \\& \qquad {} +\frac{q^{1-p}}{ [ 1-p ] _{q}} \int _{0}^{a}x^{1-p}D_{q}h_{q,r,a}f ( x ) \Biggl( \sum_{i=0}^{p-1} \bigl[ h_{q,r,a}f ( x ) \bigr] ^{p-1-i} \bigl[ h_{q,r,a}f ( qx ) \bigr] ^{i} \Biggr) \,d_{q}x \\& \quad = \bigl( h_{q,r,a}f ( b ) \bigr) ^{p} \frac{b^{1-p}}{ [ 1-p ] _{q}} \\& \qquad {} -\frac{q^{1-p}}{ [ 1-p ] _{q}} \int _{a}^{b}x^{1-p}D_{q}h_{q,r,a}f ( x ) \Biggl( \sum_{i=0}^{p-1} \bigl[ h_{q,r,a}f ( x ) \bigr] ^{p-1-i} \bigl[ h_{q,r,a}f ( qx ) \bigr] ^{i} \Biggr) \,d_{q}x. \end{aligned}$$

We notice that from (2.2)

$$ \bigl( h_{q,r,a}f ( b ) \bigr) ^{p} \frac{b^{1-p}}{ [ 1-p ] _{q}}=-q^{p-1} \bigl( h_{q,r,a}f ( b ) \bigr) ^{p} \frac{b^{1-p}}{ [ p-1 ] _{q}} $$

is negative since \(p-1\in \mathbb{N} \), \(p-1>0\) and \(h_{q,r,a}f ( b ) >0\) with \(b>0\). Also, from the definition of \(h_{q,r,a}f ( x ) \) we have

$$\begin{aligned}& D_{q}h_{q,r,a}f ( x ) \\& \quad = D_{q} \biggl( \frac{1}{r ( x ) } \int _{a}^{x}r ( t ) f ( t ) \,d_{q}t \biggr) \\& \quad = D_{q} \biggl( \frac{1}{r ( x ) } \int _{0}^{x}r ( t ) f ( t ) \,d_{q}t \biggr) -D_{q} \biggl( \frac{1}{r ( x ) }\int _{0}^{a}r ( t ) f ( t ) \,d_{q}t \biggr) \\& \quad = \frac{1}{r ( qx ) }D_{q} \biggl( \int _{0}^{x}r ( t ) f ( t ) \,d_{q}t \biggr) + \biggl( \int _{0}^{x}r ( t ) f ( t ) \,d_{q}t \biggr) D_{q} \frac{1}{r ( x ) }- \biggl( \int _{0}^{a}r ( t ) f ( t ) \,d_{q}t \biggr) D_{q}\frac{1}{r ( x ) } \\& \quad = \frac{1}{r ( qx ) }D_{q} \biggl( \int _{0}^{x}r ( t ) f ( t ) \,d_{q}t \biggr) + \biggl( \int _{a}^{x}r ( t ) f ( t ) \,d_{q}t \biggr) D_{q} \frac{1}{r ( x ) } \\& \quad = \frac{r ( x ) }{r ( qx ) }f ( x ) + \biggl( \int _{a}^{x}r ( t ) f ( t ) \,d_{q}t \biggr) D_{q}\frac{1}{r ( x ) } \\& \quad = \frac{r ( x ) }{r ( qx ) }f ( x ) -h_{q,r,a}f ( x ) \frac{D_{q}r ( x ) }{r ( qx ) }. \end{aligned}$$

Hence, by \([ 1-p ] _{q}=-\frac{1}{q^{ ( p-1 ) }} [ ( p-1 ) ] _{q}\)

$$\begin{aligned}& [ p-1 ] _{q} \int _{a}^{b} \bigl( H_{q,r,a}f ( x ) \bigr) ^{p}\,d_{q}x \\& \quad \leq \int _{a}^{b}x^{1-p} \biggl( \frac{r ( x ) }{r ( qx ) }f ( x ) -h_{r,a}f ( x ) \frac{D_{q}r ( x ) }{r ( qx ) } \biggr) \Biggl( \sum_{i=0}^{p-1} \bigl[ h_{q,r,a}f ( x ) \bigr] ^{p-1-i} \bigl[ h_{q,r,a}f ( qx ) \bigr] ^{i} \Biggr) \,d_{q}x \\& \quad = \int _{a}^{b}x^{1-p} \frac{r ( x ) }{r ( qx ) }f ( x ) \bigl[ h_{q,r,a}f ( x ) \bigr] ^{p-1} \Biggl( \sum_{i=0}^{p-1} \biggl[ \frac{h_{q,r,a}f ( qx ) }{h_{q,r,a}f ( x ) } \biggr] ^{i} \Biggr) \,d_{q}x \\& \qquad {} - \int _{a}^{b}x^{1-p} \bigl[ h_{q,r,a}f ( x ) \bigr] ^{p}\frac{D_{q}r ( x ) }{r ( qx ) } \Biggl( \sum_{i=0}^{p-1} \biggl[ \frac{h_{q,r,a}f ( qx ) }{h_{q,r,a}f ( x ) } \biggr] ^{i} \Biggr) \,d_{q}x, \end{aligned}$$

or equivalently

$$\begin{aligned}& \int _{a}^{b} \Biggl[ [ p-1 ] _{q}+x \frac{D_{q}r ( x ) }{r ( qx ) } \Biggl( \sum _{i=0}^{p-1} \biggl[ \frac{h_{q,r,a}f ( qx ) }{h_{q,r,a}f ( x ) } \biggr] ^{i} \Biggr) \Biggr] \bigl( H_{q,r,a}f ( x ) \bigr) ^{p}\,d_{q}x \\& \quad \leq \int _{a}^{b} \Biggl( \frac{r ( x ) }{r ( qx ) }\sum_{i=0}^{p-1} \biggl[ \frac{h_{q,r,a}f ( qx ) }{h_{q,r,a}f ( x ) } \biggr] ^{i} \Biggr) f ( x ) \bigl( H_{q,r,a}f ( x ) \bigr) ^{p-1}\,d_{q}x. \end{aligned}$$

Now, using (4.5) and the q-Hölder inequality, we have

$$\begin{aligned}& \frac{p}{\lambda } \int _{a}^{b} \bigl( H_{q,r,a}f ( x ) \bigr) ^{p}\,d_{q}x \\& \quad \leq \Biggl( \int _{a}^{b} \Biggl( \frac{r ( x ) }{r ( qx ) }\sum _{i=0}^{p-1} \biggl[ \frac{h_{q,r,a}f ( qx ) }{h_{q,r,a}f ( x ) } \biggr] ^{i} \Biggr) ^{p}f^{p} ( x ) \,d_{q}x \Biggr) ^{\frac{1}{p}} \biggl( \int _{a}^{b} \bigl[ H_{q,r,a}f ( x ) \bigr] ^{ ( p-1 ) p^{ \prime }}\,d_{q}x \biggr) ^{\frac{1}{p^{\prime }}}, \end{aligned}$$

where \(\frac{1}{p}+\frac{1}{p^{\prime }}=1\), that is,

$$ \int _{a}^{b} \bigl( H_{q,r,a}f ( x ) \bigr) ^{p}\,d_{q}x \leq \frac{\lambda ^{p}}{p^{p}} \int _{0}^{\infty } \Biggl( \frac{r ( x ) }{r ( qx ) }\sum _{i=0}^{p-1} \biggl[ \frac{h_{q,r,a}f ( qx ) }{h_{q,r,a}f ( x ) } \biggr] ^{i} \Biggr) ^{p}f^{p} ( x ) \,d_{q}x. $$

If we take \(c>a\), then

$$\begin{aligned} \int _{c}^{b} \bigl( H_{q,r,a}f ( x ) \bigr) ^{p}\,d_{q}x \leq& \int _{a}^{b} \bigl( H_{q,r,a}f ( x ) \bigr) ^{p}\,d_{q}x \\ \leq& \frac{\lambda ^{p}}{p^{p}} \int _{0}^{\infty } \Biggl( \frac{r ( x ) }{r ( qx ) }\sum _{i=0}^{p-1} \biggl[ \frac{h_{q,r,a}f ( qx ) }{h_{q,r,a}f ( x ) } \biggr] ^{i} \Biggr) ^{p}f^{p} ( x ) \,d_{q}x. \end{aligned}$$

Invoking the dominated convergence theorem, taking \(a\rightarrow \infty \), we get

$$ \int _{c}^{b} \bigl( H_{q,r}f ( x ) \bigr) ^{p}\,d_{q}x \leq \frac{\lambda ^{p}}{p^{p}} \int _{0}^{\infty } \Biggl( \frac{r ( x ) }{r ( qx ) }\sum _{i=0}^{p-1} \biggl[ \frac{h_{q,r,a}f ( qx ) }{h_{q,r,a}f ( x ) } \biggr] ^{i} \Biggr) ^{p}f^{p} ( x ) \,d_{q}x $$

for all \(c,b>0\). Finally, letting \(b\rightarrow \infty \) and \(c\rightarrow 0\),

$$ \int _{0}^{\infty } \bigl( H_{q,r}f ( x ) \bigr) ^{p}\,d_{q}x \leq \frac{\lambda ^{p}}{p^{p}} \int _{0}^{\infty } \Biggl( \frac{r ( x ) }{r ( qx ) }\sum _{i=0}^{p-1} \biggl[ \frac{h_{q,r,a}f ( qx ) }{h_{q,r,a}f ( x ) } \biggr] ^{i} \Biggr) ^{p}f^{p} ( x ) \,d_{q}x. $$

 □

In Theorem 6 if we take the limit \(q\rightarrow 1^{-}\) we obtain the following theorem, proved by N. Levinson in 1964 (cf. [13, Theorem 4]).

Remark 3

Let f be a nonnegative function on \(( 0,\infty ) \), r being absolutely continuous function on \(( 0,\infty ) \) and \(p>1\). Also assume \(\int _{0}^{\infty } ( f ( x ) ) ^{p}\,dx\) is convergent, and

$$ \frac{p-1}{p}+x\frac{r^{\prime }}{r}\geq \frac{1}{\lambda }, $$

for almost every \(x>0\) and for some \(\lambda >0\). Then we have the following inequality:

$$ \int _{0}^{\infty } \bigl( H_{r}f ( x ) \bigr) ^{p}\,dx \leq \lambda ^{p} \int _{0}^{\infty }f^{p} ( x ) \,dx, $$

where

$$ H_{r}f ( x ) =\frac{1}{xr ( x ) } \int _{0}^{x}r ( t ) f ( t ) \,dt. $$

Theorem 7

Assume f is a nonnegative function on \(( 0,\infty ) \), u is absolutely continuous function on \(( 0,\infty ) \) and \(p>1\). Also assume \(\int _{a}^{b} ( f ( x ) ) ^{p}\,d_{q}x\) is convergent, and

$$ \frac{ [ p-1 ] _{q}}{p}-\frac{x}{p} \frac{D_{q}u ( x ) }{u ( x ) }\sum _{i=0}^{p-1} \biggl( \frac{u ( qx ) }{u ( x ) } \biggr) ^{\frac{i}{p}}\sum_{i=0}^{p-1} \biggl[ \frac{h_{q,r,a}g ( qx ) }{h_{q,r,a}g ( x ) } \biggr] ^{i} \geq \frac{1}{\lambda }, $$
(4.6)

for almost every \(x>0\) and for some \(\lambda >0\). Then we have the following inequality:

$$ \int _{0}^{\infty } \bigl( H_{q}f ( x ) \bigr) ^{p}u ( x ) \,d_{q}x\leq \frac{\lambda ^{p}}{p^{p}} \int _{0}^{ \infty } \Biggl( \sum _{i=0}^{p-1} \biggl[ \frac{h_{q,r,a}g ( qx ) }{h_{q,r,a}g ( x ) } \biggr] ^{i} \Biggr) ^{p}f^{p} ( x ) u ( qx ) \,d_{q}x, $$
(4.7)

where

$$ H_{q}f ( x ) =\frac{1}{x} \int _{0}^{x}f ( t ) \,d_{q}t. $$

Proof

If we consider \(r ( x ) = ( \frac{1}{u ( x ) } ) ^{ \frac{1}{p}}\), then

$$ f ( x ) =r ( x ) g ( x ) = \biggl( \frac{1}{u ( x ) } \biggr) ^{\frac{1}{p}}g ( x ) $$

and we apply Theorem 6 to g, we assume \(0< a< b<\infty \) and

$$ h_{q,r,a}g ( x ) =\frac{1}{r ( x ) } \int _{a}^{x}r ( t ) g ( t ) \,d_{q}t= \bigl( u ( x ) \bigr) ^{\frac{1}{p}} \int _{a}^{x}f ( t ) \,d_{q}t. $$

Then, defining \(H_{q,r,a}g ( x ) =\frac{1}{x}h_{q,r,a}g ( x ) \), and integrating by parts from (2.5) with \(w= ( h_{q,r,a}g ( x ) ) ^{p}\) and \(D_{q}v ( x ) =x^{-p}\) noting that \(v(x)=\frac{x^{1-p}}{ [ 1-p ] _{q}}\) we get

$$\begin{aligned}& \int _{a}^{b} \bigl( H_{q,r,a}g ( x ) \bigr) ^{p}\,d_{q}x \\& \quad = \bigl( h_{q,r,a}g ( b ) \bigr) ^{p} \frac{b^{1-p}}{ [ 1-p ] _{q}} \\& \qquad {} -\frac{q^{1-p}}{ [ 1-p ] _{q}} \int _{0}^{b}x^{1-p}D_{q}h_{q,r,a}g ( x ) \Biggl( \sum_{i=0}^{p-1} \bigl[ h_{q,r,a}g ( x ) \bigr] ^{p-1-i} \bigl[ h_{q,r,a}g ( qx ) \bigr] ^{i} \Biggr) \,d_{q}x \\& \qquad {} +\frac{q^{1-p}}{ [ 1-p ] _{q}} \int _{0}^{a}x^{1-p}D_{q}h_{q,r,a}g ( x ) \Biggl( \sum_{i=0}^{p-1} \bigl[ h_{q,r,a}g ( x ) \bigr] ^{p-1-i} \bigl[ h_{q,r,a}g ( qx ) \bigr] ^{i} \Biggr) \,d_{q}x \\& \quad = \bigl( h_{q,r,a}g ( b ) \bigr) ^{p} \frac{b^{1-p}}{ [ 1-p ] _{q}} \\& \qquad {} -\frac{q^{1-p}}{ [ 1-p ] _{q}} \int _{a}^{b}x^{1-p}D_{q}h_{q,r,a}g ( x ) \Biggl( \sum_{i=0}^{p-1} \bigl[ h_{q,r,a}g ( x ) \bigr] ^{p-1-i} \bigl[ h_{q,r,a}g ( qx ) \bigr] ^{i} \Biggr) \,d_{q}x. \end{aligned}$$

We notice that from (2.2)

$$ \bigl( h_{q,r,a}g ( b ) \bigr) ^{p} \frac{b^{1-p}}{ [ 1-p ] _{q}}=-q^{p-1} \bigl( h_{q,r,a}g ( b ) \bigr) ^{p} \frac{b^{1-p}}{ [ p-1 ] _{q}} $$

is negative since \(p-1\in \mathbb{N} \), \(p-1>0\) and \(h_{q,r,a}g ( b ) >0\) with \(b>0\). Also, from the definition of \(h_{q,r,a}g ( x ) \) we have

$$\begin{aligned}& D_{q}h_{q,r,a}g ( x ) \\& \quad = D_{q} \biggl( \bigl( u ( x ) \bigr) ^{\frac{1}{p}} \int _{a}^{x}f ( t ) \,d_{q}t \biggr) \\& \quad = D_{q} \biggl( \bigl( u ( x ) \bigr) ^{\frac{1}{p}} \int _{0}^{x}f ( t ) \,d_{q}t \biggr) -D_{q} \biggl( \bigl( u ( x ) \bigr) ^{\frac{1}{p}} \int _{0}^{a}f ( t ) \,d_{q}t \biggr) \\& \quad = \bigl( u ( qx ) \bigr) ^{\frac{1}{p}}D_{q} \biggl( \int _{0}^{x}f ( t ) \,d_{q}t \biggr) + \biggl( \int _{0}^{x}f ( t ) \,d_{q}t \biggr) D_{q} \bigl( u ( x ) \bigr) ^{\frac{1}{p}}- \biggl( \int _{0}^{a}f ( t ) \,d_{q}t \biggr) D_{q} \bigl( u ( x ) \bigr) ^{\frac{1}{p}} \\& \quad = \bigl( u ( qx ) \bigr) ^{\frac{1}{p}}D_{q} \biggl( \int _{0}^{x}f ( t ) \,d_{q}t \biggr) + \biggl( \int _{a}^{x}f ( t ) \,d_{q}t \biggr) D_{q} \bigl( u ( x ) \bigr) ^{\frac{1}{p}} \\& \quad = \bigl( u ( qx ) \bigr) ^{\frac{1}{p}}f ( x ) + \frac{h_{q,r,a}g ( x ) }{ ( u ( x ) ) ^{\frac{1}{p}}}\frac{D_{q}u ( x ) }{\sum_{i=0}^{p-1} ( u ( x ) ) ^{\frac{p-1-i}{p}} ( u ( qx ) ) ^{\frac{i}{p}}} \\& \quad = \bigl( u ( qx ) \bigr) ^{\frac{1}{p}}f ( x ) +\frac{h_{q,r,a}g ( x ) }{u ( x ) }D_{q}u ( x ) \sum_{i=0}^{p-1} \biggl( \frac{u ( qx ) }{u ( x ) } \biggr) ^{\frac{i}{p}}. \end{aligned}$$

Hence, by \([ 1-p ] _{q}=-\frac{1}{q^{ ( p-1 ) }} [ ( p-1 ) ] _{q}\)

$$\begin{aligned}& [ p-1 ] _{q} \int _{a}^{b} \bigl( H_{q,r,a}g ( x ) \bigr) ^{p}\,d_{q}x \\& \quad \leq \int _{a}^{b}x^{1-p} \bigl( u ( qx ) \bigr) ^{\frac{1}{p}}f ( x ) \bigl[ h_{q,r,a}g ( x ) \bigr] ^{p-1} \sum_{i=0}^{p-1} \biggl[ \frac{h_{q,r,a}g ( qx ) }{h_{q,r,a}g ( x ) } \biggr] ^{i}\,d_{q}x \\& \qquad {} + \int _{a}^{b}x^{1-p} \frac{ ( h_{q,r,a}g ( x ) ) ^{p}}{u ( x ) }D_{q}u ( x ) \sum_{i=0}^{p-1} \biggl( \frac{u ( qx ) }{u ( x ) } \biggr) ^{ \frac{i}{p}}\sum _{i=0}^{p-1} \biggl[ \frac{h_{q,r,a}g ( qx ) }{h_{q,r,a}g ( x ) } \biggr] ^{i}\,d_{q}x \end{aligned}$$

or equivalently

$$\begin{aligned}& \int _{a}^{b} \Biggl[ [ p-1 ] _{q}-x \frac{D_{q}u ( x ) }{u ( x ) }\sum_{i=0}^{p-1} \biggl( \frac{u ( qx ) }{u ( x ) } \biggr) ^{\frac{i}{p}}\sum _{i=0}^{p-1} \biggl( \frac{h_{q,r,a}g ( qx ) }{h_{q,r,a}g ( x ) } \biggr) ^{i} \Biggr] \bigl( H_{q,r,a}g ( x ) \bigr) ^{p}\,d_{q}x \\& \quad \leq \int _{a}^{b} \bigl( u ( qx ) \bigr) ^{ \frac{1}{p}}\sum_{i=0}^{p-1} \biggl[ \frac{h_{q,r,a}g ( qx ) }{h_{q,r,a}g ( x ) } \biggr] ^{i}f ( x ) \bigl[ H_{q,r,a}g ( x ) \bigr] ^{p-1}\,d_{q}x. \end{aligned}$$

Finally, by using (4.6) and the q-Hölder inequality, we have

$$ \int _{0}^{\infty } \bigl( H_{q,r}f ( x ) \bigr) ^{p}\,d_{q}x \leq \frac{\lambda ^{p}}{p^{p}} \int _{0}^{\infty } \Biggl( \sum _{i=0}^{p-1} \biggl[ \frac{h_{q,r,a}g ( qx ) }{h_{q,r,a}g ( x ) } \biggr] ^{i} \Biggr) ^{p}f^{p} ( x ) u ( qx ) \,d_{q}x $$

and

$$ \int _{0}^{\infty } \bigl( H_{q}f ( x ) \bigr) ^{p}u ( x ) \,d_{q}x\leq \frac{\lambda ^{p}}{p^{p}} \int _{0}^{ \infty } \Biggl( \sum _{i=0}^{p-1} \biggl[ \frac{h_{q,r,a}g ( qx ) }{h_{q,r,a}g ( x ) } \biggr] ^{i} \Biggr) ^{p}f^{p} ( x ) u ( qx ) \,d_{q}x, $$

and this completes the proof. □

In Theorem 7 if we take the limit \(q\rightarrow 1^{-}\) we obtain the following result, proved by N. Levinson in 1964 [13] on continuous analysis.

Remark 4

Assume that f is a nonnegative function on \(( 0,\infty ) \), u is absolutely continuous function on \(( 0,\infty )\), and \(p>1\). Also assume \(\int _{a}^{b} ( f ( x ) ) ^{p}\,dx\) is convergent, and

$$ \frac{p-1}{p}-px\frac{u^{\prime }}{u}\geq \frac{1}{\lambda }, $$

for almost every \(x>0\) and for some \(\lambda >0\). Then we have the following inequality:

$$ \int _{0}^{\infty } \bigl( Hf ( x ) \bigr) ^{p}u ( x ) \,dx\leq \lambda ^{p} \int _{0}^{\infty }f^{p} ( x ) u ( x ) \,dx, $$

where

$$ Hf ( x ) =\frac{1}{x} \int _{0}^{x}f ( t ) \,dt. $$

Availability of data and materials

Data sharing not applicable to this paper as no data sets were generated or analysed during the current study.

References

  1. AlNemer, G., Zakarya, M., El-Hamid, A., Hoda, A., Agarwal, P., Rezk, H.M.: Some dynamic Hilbert-type inequalities on time scales. Symmetry 12(9), 1410 (2020)

    Article  Google Scholar 

  2. Ernst, T.: A Comprehensive Treatment of q-Calculus. Springer, Basel (2012)

    Book  Google Scholar 

  3. Gauchman, H.: Integral inequalities in q-calculus. Comput. Math. Appl. 47, 281–300 (2004)

    Article  MathSciNet  Google Scholar 

  4. Hardy, G.H.: Note on a theorem of Hilbert. Math. Z. 6, 314–317 (1920)

    Article  MathSciNet  Google Scholar 

  5. Izumi, M., Izumi, S., Peterson, G.: On Hardy’s inequality and its generalization. Tohoku Math. J. 21, 601–613 (1999)

    MathSciNet  MATH  Google Scholar 

  6. Jackson, F.H.: On a q-definite integrals. Q. J. Pure Appl. Math. 41, 193–203 (1910)

    MATH  Google Scholar 

  7. Jain, S., Mehrez, K., Baleanu, D., Agarwal, P.: Certain Hermite–Hadamard inequalities for logarithmically convex functions with applications. Mathematics 7(2), 163 (2019)

    Article  MathSciNet  Google Scholar 

  8. Jhanthanam, S., Tariboon, J., Ntouyas, S.K., Nonlaopon, K.: On q-Hermite–Hadamard inequalities for differentiable convex functions. Mathematics 7, 632 (2019)

    Article  Google Scholar 

  9. Kac, V., Cheung, P.: Quantum Calculus. Springer, Berlin (2001)

    MATH  Google Scholar 

  10. Kadakal, M., Işcan, I., Agarwal, P., Jleli, M.: Exponential trigonometric convex functions and Hermite–Hadamard type inequalities. Math. Slovaca 71(1), 43–56 (2021)

    Article  MathSciNet  Google Scholar 

  11. Kufner, A., Maligranda, L., Persson, L.E.: The Hardy Inequality—About Its History and Some Related Results. Vydavatelsky Servis Publishing House, Pilsen (2007)

    MATH  Google Scholar 

  12. Kufner, A., Persson, L.E.: Weighted Inequalities of Hardy Type. World Scientific, Singapore (2003)

    Book  Google Scholar 

  13. Levinson, N.: Generalizations of an inequality of Hardy. Duke Math. J. 31, 389–394 (1964)

    MathSciNet  MATH  Google Scholar 

  14. Mehrez, K., Agarwal, P.: New Hermite–Hadamard type integral inequalities for convex functions and their applications. J. Comput. Appl. Math. 350, 274–285 (2019)

    Article  MathSciNet  Google Scholar 

  15. Moazzena, A., Lashkaripour, R.: Some new extensions of Hardy’s inequality. Int. J. Nonlinear Anal. Appl. 5(1), 98–109 (2014)

    MATH  Google Scholar 

  16. Mohammed, P.O., Abdeljawad, T., Baleanu, D., Kashuri, A., Hamasalh, F., Agarwal, P.: New fractional inequalities of Hermite–Hadamard type involving the incomplete gamma functions. J. Inequal. Appl. 2020, 263 (2020)

    Article  MathSciNet  Google Scholar 

  17. Prabseang, J., Nonlaopon, K., Ntouyas, S.K.: On the refinemenet of quantum Hermite–Hadamard inequalities for continuous convex functions. J. Math. Inequal. 14(3), 875–885 (2020)

    Article  MathSciNet  Google Scholar 

  18. Saker, S.H., O’Regan, D., Kenawy, M.R., Agarwal, R.P.: Fractional Hardy type inequalities via conformable calculus. Mem. Differ. Equ. Math. Phys. 73, 131–140 (2018)

    MathSciNet  MATH  Google Scholar 

  19. Sarikaya, M.Z., Yildirim, H.: Some Hardy-type integral inequalities. J. Inequal. Pure Appl. Math. 7(5), Art 178 (2006)

    MathSciNet  MATH  Google Scholar 

  20. Tomar, M., Agarwal, P., Choi, J.: Hermite–Hadamard type inequalities for generalized convex functions on fractal sets style. Bol. Soc. Parana. Mat. 38(1), 101–116 (2020)

    Article  MathSciNet  Google Scholar 

Download references

Acknowledgements

The authors would like to express their sincere thanks to the editor and the anonymous reviewers for their helpful comments and suggestions.

Funding

There is no funding.

Author information

Authors and Affiliations

Authors

Contributions

All authors contributed equally to the writing of this paper. All authors read and approved the final manuscript.

Corresponding author

Correspondence to Necmettin Alp.

Ethics declarations

Competing interests

The authors declare that they have no competing interests.

Rights and permissions

Open Access This article is licensed under a Creative Commons Attribution 4.0 International License, which permits use, sharing, adaptation, distribution and reproduction in any medium or format, as long as you give appropriate credit to the original author(s) and the source, provide a link to the Creative Commons licence, and indicate if changes were made. The images or other third party material in this article are included in the article’s Creative Commons licence, unless indicated otherwise in a credit line to the material. If material is not included in the article’s Creative Commons licence and your intended use is not permitted by statutory regulation or exceeds the permitted use, you will need to obtain permission directly from the copyright holder. To view a copy of this licence, visit http://creativecommons.org/licenses/by/4.0/.

Reprints and permissions

About this article

Check for updates. Verify currency and authenticity via CrossMark

Cite this article

Alp, N., Sarikaya, M.Z. q-Hardy type inequalities for quantum integrals. Adv Differ Equ 2021, 355 (2021). https://doi.org/10.1186/s13662-021-03514-6

Download citation

  • Received:

  • Accepted:

  • Published:

  • DOI: https://doi.org/10.1186/s13662-021-03514-6

MSC

Keywords